Bạn chưa đăng nhập. Vui lòng đăng nhập để hỏi bài

Những câu hỏi liên quan
Blue Moon
Xem chi tiết
tth_new
11 tháng 8 2020 lúc 20:12

Ta còn có:

Bất đẳng thức \(\frac{1}{a\left(a+b\right)}+\frac{1}{b\left(b+c\right)}+\frac{1}{c\left(c+a\right)}\ge\frac{1}{k\left(a^2+b^2+c^2\right)+\left(\frac{2}{9}-k\right)\left(ab+bc+ca\right)}\)

đúng với mọi a,b,c,k không âm (k = \(\text{constant}\))

Khách vãng lai đã xóa
Rhider
Xem chi tiết
Rhider
19 tháng 12 2021 lúc 20:14

ai giỏi ạ

Mai Tuấn Hưng
Xem chi tiết
Nguyễn Đăng Nhân
22 tháng 2 2022 lúc 17:24

Áp dụng đánh giá \(x^2+y^2+z^2\ge\frac{1}{3}\left(x+y+z\right)^2\) , ta được:

\(\left(\frac{a}{b+2c}\right)^2+\left(\frac{b}{c+2a}\right)^2+\left(\frac{c}{a+2b}\right)^2\ge\frac{1}{3}\left(\frac{a}{b+2c}+\frac{b}{c+2a}+\frac{c}{a+2b}\right)\)

Vậy ta cần chứng minh:

\(\frac{a}{b+2c}+\frac{b}{c+2a}+\frac{c}{a+2b}\ge1\)

Áp dụng bất đẳng thức Bunhiacopxki dạng phân thức ta được:

\(\frac{a}{b+2c}+\frac{b}{c+2a}+\frac{c}{a+2b}\ge\frac{\left(a+b+c\right)^2}{3\left(ab+bc+ca\right)}\)

Vậy theo đánh giá ta được: \(\left(a+b+c\right)^2\ge3\left(ab+bc+ca\right)\), do đó ta được:

\(\frac{a}{b+2c}+\frac{b}{c+2a}+\frac{c}{a+2b}\ge1\)

Vậy bất đẳng thức ban đầu được chứng minh.

Khách vãng lai đã xóa
l҉o҉n҉g҉ d҉z҉
Xem chi tiết
Đặng Ngọc Quỳnh
1 tháng 1 2021 lúc 8:42

giả sử \(a\ge b\ge c\ge0\)

Ta có: \(a+\frac{b}{2}-\frac{a^2+ab+b^2}{a+b}=\frac{1}{2}\left(ab-b^2\right)\ge0\Rightarrow a+\frac{b}{2}\ge\frac{a^2+ab+b^2}{a+b}\)

\(b+\frac{a}{2}-\frac{a^2+ab+b^2}{a+b}=\frac{1}{2}\left(ab-a^2\right)\le0\Rightarrow b+\frac{a}{2}\le\frac{a^2+ab+b^2}{a+b}\)

Tương tự: \(b+\frac{c}{2}\ge\frac{b^2+bc+c^2}{b+c}\ge c+\frac{b}{2};a+\frac{c}{2}\ge\frac{a^2+ac+c^2}{a+c}\ge c+\frac{a}{2}\)

Lại có:+) \(\frac{a^3-b^3}{a+b}+\frac{b^3-c^3}{b+c}+\frac{c^3-a^3}{c+a}\)

\(=\left(a-b\right)\frac{a^2+ab+b^2}{a+b}+\left(b-c\right)\frac{b^2+bc+c^2}{b+c}-\left(a-c\right)\frac{a^2+ac+c^2}{a+c}\)

\(\ge\left(a-b\right)\left(b+\frac{a}{2}\right)+\left(b-c\right)\left(c+\frac{a}{2}\right)-\left(a-c\right)\left(a+\frac{c}{2}\right)\)

\(\ge\frac{-1}{4}\left[\left(a-b\right)^2+\left(b-c\right)^2+\left(c-a\right)^2\right]\left(1\right)\)

+) \(\frac{a^3-b^3}{a+b}+\frac{b^3-c^3}{b+c}+\frac{c^3-a^3}{c+a}\)

\(=\left(a-b\right)\frac{a^2+ab+b^2}{a+b}+\left(b-c\right)\frac{b^2+bc+c^2}{b+c}-\left(a-c\right)\frac{a^2+ac+c^2}{a+c}\)

\(\le\left(a-b\right)\left(a+\frac{b}{2}\right)+\left(b-c\right)\left(b+\frac{c}{2}\right)-\left(a-c\right)\left(c+\frac{a}{2}\right)\)

\(\le\frac{1}{4}\left[\left(a-b\right)^2+\left(b-c\right)^2+\left(c-a\right)^2\right]\left(2\right)\)

Từ 1,2 => đpcm

Khách vãng lai đã xóa
tth_new
2 tháng 1 2021 lúc 14:46

BĐT đã cho tuong duong voi:

\(\left|\frac{\left(a-b\right)\left(b-c\right)\left(c-a\right)\left(ab+bc+ca\right)}{\left(a+b\right)\left(b+c\right)\left(c+a\right)}\right|\le\frac{1}{4}\left[\Sigma\left(a-b\right)^2\right]\)

Theo AM-GM ta có: \(\left(ab+bc+ca\right)\le\frac{9}{8}\cdot\frac{\left(a+b\right)\left(b+c\right)\left(c+a\right)}{a+b+c}\)

Có: \(VT\le\frac{9}{8}\left|\frac{\sqrt{\left(a-b\right)^2\left(b-c\right)^2\left(c-a\right)^2}}{\left(a+b+c\right)}\right|=\frac{9\sqrt{\left(a-b\right)^2\left(b-c\right)^2\left(c-a\right)^2}}{8\left(a+b+c\right)}\)

Cần chứng minh: \(4\left(a+b+c\right)^2\left[\Sigma\left(a-b\right)^2\right]^2\ge9\left(a-b\right)^2\left(b-c\right)^2\left(c-a\right)^2\)

Rõ ràng \(\Sigma\left(a-b\right)^2\ge3\sqrt[3]{\left(a-b\right)^2\left(b-c\right)^2\left(c-a\right)^2}\)

Cần cm: \(36\left(a+b+c\right)^2\sqrt[3]{\left(a-b\right)^4\left(b-c\right)^4\left(c-a\right)^4}\ge9\sqrt[3]{\left(a-b\right)^6\left(b-c\right)^6\left(c-a\right)^6}\)

Hay \(4\left(a+b+c\right)^2\ge\sqrt[3]{\left(a-b\right)^2\left(b-c\right)^2\left(c-a\right)^2}\)

Tiếp tục là điều hiển nhiên do \(VT\ge4\left[\left(a+b+c\right)^2-3\left(ab+bc+ca\right)\right]\)

\(=2\left[\left(a-b\right)^2+\left(b-c\right)^2+\left(c-a\right)^2\right]\)

\(\ge6\sqrt[3]{\left(a-b\right)^2\left(b-c\right)^2\left(c-a\right)^2}\ge VP\)

Đẳng thức xảy ra khi \(\hept{\begin{cases}\left(a-b\right)\left(b-c\right)\left(c-a\right)=0\\a-b=b-c=c-a\\a=b=c\end{cases}}\Leftrightarrow a=b=c.\)

Khách vãng lai đã xóa
Rhider
Xem chi tiết
Akai Haruma
19 tháng 12 2021 lúc 20:39

Lời giải:

Dấu "=" không xảy ra.
Áp dụng BĐT AM-GM:

\(\text{VT}\leq \frac{a+(b+1)}{2}+\frac{b+(c+1)}{2}+\frac{c+(a+1)}{2}=\frac{2(a+b+c)+3}{2}\)

\(< \frac{3(a+b+c+ab+bc+ac+abc+1)}{2}=\frac{3(a+1)(b+1)(c+1)}{2}\)

Ta có đpcm.

Akai Haruma
19 tháng 12 2021 lúc 20:40

Lần sau bạn lưu ý đăng 1 bài 1 lần thôi. Đăng nhiều lần coi như spam và sẽ bị xóa không thương tiếc đấy nhé.

Lê Song Phương
Xem chi tiết
Nguyễn Trang Anh
19 tháng 12 2021 lúc 20:08

mk lớp 7

Khách vãng lai đã xóa
Flower in Tree
19 tháng 12 2021 lúc 20:41

Dấu '' = '' không xảy ra

Áp dụng BĐT AM-GM:

Dấu "=" không xảy ra.
Áp dụng BĐT AM-GM:

\(\text{VT}\leq \frac{a+(b+1)}{2}+\frac{b+(c+1)}{2}+\frac{c+(a+1)}{2}=\frac{2(a+b+c)+3}{2}\)

\(< \frac{3(a+b+c+ab+bc+ac+abc+1)}{2}=\frac{3(a+1)(b+1)(c+1)}{2}\)

Ta có đpcm.

Khách vãng lai đã xóa
Lê Song Phương
19 tháng 12 2021 lúc 21:11

Em thưa anh:

Áp dụng BĐT Cô-si cho hai số dương \(\frac{a}{a+1}\)và \(\frac{1}{c+1}\), ta có:

\(\sqrt{\frac{a}{\left(a+1\right)\left(c+1\right)}}=\sqrt{\frac{a}{a+1}.\frac{1}{c+1}}\le\frac{1}{2}\left(\frac{a}{a+1}+\frac{1}{c+1}\right)\)

Tương tự, ta có: \(\sqrt{\frac{b}{\left(a+1\right)\left(b+1\right)}}\le\frac{1}{2}\left(\frac{1}{a+1}+\frac{b}{b+1}\right)\)

\(\sqrt{\frac{c}{\left(c+1\right)\left(b+1\right)}}\le\frac{1}{2}\left(\frac{c}{c+1}+\frac{1}{b+1}\right)\)

Công vế theo vế, ta có: \(\sqrt{\frac{a}{\left(a+1\right)\left(c+1\right)}}+\sqrt{\frac{b}{\left(b+1\right)\left(a+1\right)}}+\sqrt{\frac{c}{\left(c+1\right)\left(b+1\right)}}\)

\(\le\frac{1}{2}\left(\frac{a}{a+1}+\frac{1}{c+1}+\frac{b}{b+1}+\frac{1}{a+1}+\frac{c}{c+1}+\frac{1}{b+1}\right)\)

\(=\frac{1}{2}\left(\frac{a+1}{a+1}+\frac{b+1}{b+1}+\frac{c+1}{c+1}\right)=\frac{1}{2}\left(1+1+1\right)=\frac{3}{2}\)

\(\Leftrightarrow\sqrt{\frac{a}{\left(a+1\right)\left(c+1\right)}}+\sqrt{\frac{b}{\left(b+1\right)\left(a+1\right)}}+\sqrt{\frac{c}{\left(c+1\right)\left(b+1\right)}}\le\frac{3}{2}\)

Nhân cả hai vế với \(\left(a+1\right)\left(b+1\right)\left(c+1\right)\)(vì a,b,c>0 nên BĐT lúc này không đổi chiều), ta có đpcm.

Khách vãng lai đã xóa
pham thi thu trang
Xem chi tiết
Tùng Nguyễn
Xem chi tiết
Thắng Nguyên
Xem chi tiết
Kiệt Nguyễn
28 tháng 8 2020 lúc 9:36

Áp dụng giả thiết và một đánh giá quen thuộc, ta được: \(16\left(a+b+c\right)\ge\frac{1}{a}+\frac{1}{b}+\frac{1}{c}=\frac{ab+bc+ca}{abc}=\frac{\left(ab+bc+ca\right)^2}{abc\left(ab+bc+ca\right)}\ge\frac{3\left(a+b+c\right)}{ab+bc+ca}\)hay \(\frac{1}{6\left(ab+bc+ca\right)}\le\frac{8}{9}\)

Đến đây, ta cần chứng minh \(\frac{1}{\left(a+b+\sqrt{2\left(a+c\right)}\right)^3}+\frac{1}{\left(b+c+\sqrt{2\left(b+a\right)}\right)^3}+\frac{1}{\left(c+a+\sqrt{2\left(c+b\right)}\right)^3}\le\frac{1}{6\left(ab+bc+ca\right)}\)

 Áp dụng bất đẳng thức Cauchy cho ba số dương ta có \(a+b+\sqrt{2\left(a+c\right)}=a+b+\sqrt{\frac{a+c}{2}}+\sqrt{\frac{a+c}{2}}\ge3\sqrt[3]{\frac{\left(a+b\right)\left(a+c\right)}{2}}\)hay \(\left(a+b+\sqrt{2\left(a+c\right)}\right)^3\ge\frac{27\left(a+b\right)\left(a+c\right)}{2}\Leftrightarrow\frac{1}{\left(a+b+2\sqrt{a+c}\right)^3}\le\frac{2}{27\left(a+b\right)\left(a+c\right)}\)

Hoàn toàn tương tự ta có \(\frac{1}{\left(b+c+2\sqrt{b+a}\right)^3}\le\frac{2}{27\left(b+c\right)\left(b+a\right)}\)\(\frac{1}{\left(c+a+2\sqrt{c+b}\right)^3}\le\frac{2}{27\left(c+a\right)\left(c+b\right)}\)

Cộng theo vế các bất đẳng thức trên ta được \(\frac{1}{\left(a+b+\sqrt{2\left(a+c\right)}\right)^3}+\frac{1}{\left(b+c+\sqrt{2\left(b+a\right)}\right)^3}+\frac{1}{\left(c+a+\sqrt{2\left(c+b\right)}\right)^3}\le\frac{4\left(a+b+c\right)}{27\left(a+b\right)\left(b+c\right)\left(c+a\right)}\)Phép chứng minh sẽ hoàn tất nếu ta chỉ ra được \(\frac{4\left(a+b+c\right)}{27\left(a+b\right)\left(b+c\right)\left(c+a\right)}\le\frac{1}{6\left(ab+bc+ca\right)}\)\(\Leftrightarrow\left(a+b\right)\left(b+c\right)\left(c+a\right)\ge\frac{8}{9}\left(ab+bc+ca\right)\left(a+b+c\right)\)

Đây là một đánh giá đúng, thật vậy: đặt a + b + c = p; ab + bc + ca = q; abc = r thì bất đẳng thức trên trở thành \(pq-r\ge\frac{8}{9}pq\Leftrightarrow\frac{1}{9}pq\ge r\)*đúng vì \(a+b+c\ge3\sqrt[3]{abc}\)\(ab+bc+ca\ge3\sqrt[3]{\left(abc\right)^2}\))

Vậy bất đẳng thức được chứng minh

Đẳng thức xảy ra khi \(a=b=c=\frac{1}{4}\)

Khách vãng lai đã xóa